Themenbereiche Themenbereiche Profile Hilfe/Anleitungen Help    
Recent Posts Last 1|3|7 Days Suche Suche Tree Tree View  

Sei K ein angeordneter Körper

ZahlReich - Mathematik Hausaufgabenhilfe » ---- Archiv: Universitäts-Niveau » Analysis » Arithmetische und algebraische Grundlagen » Sei K ein angeordneter Körper « Zurück Vor »

Autor Beitrag
Seitenanfangvoriger Beitragnächster BeitragSeitenende Link zu diesem Beitrag

Sascha (Gull)
Suche alle Beiträge dieser Person in dieser Hauptrubrik
Veröffentlicht am Sonntag, den 26. November, 2000 - 14:26:   Beitrag drucken

Hi. Wer kann mir bei dieser Aufgabe helfen?

Aufgabe 3


Besten Dank.
Gruß, Sascha.
Seitenanfangvoriger Beitragnächster BeitragSeitenende Link zu diesem Beitrag

Matroid (Matroid)
Suche alle Beiträge dieser Person in dieser Hauptrubrik
Veröffentlicht am Sonntag, den 26. November, 2000 - 17:56:   Beitrag drucken

Hi Sascha,
vorausgesetzt werden die Anordnungsaxiome:
1) Für beliebige a,beK ist entweder a<b oder b<a oder a=b
2) Aus a<b und b<c folgt a<c
3) Aus a<b folgt a+c<b+c
4) Aus a<b und 0<c folgt ac<bc

Einige Hilfsaussagen:
Es gilt immer 0<xx
Beweis: Wenn x>0 dann mit Axiom 4) 0<x und 0<x =>0*x<x*x => 0<xx
Für x=1 ergibt sich insbesondere 0<1.
Weiter gilt: wenn x>0 ist auch x-1>0
Beweis: angenommen x-1<0 dann folgt aus Axiom 4), aus x-1<0 und 0<x, daß x-1*x<0. Aber das bedeutete 1<0. Widerspruch.

So nun zu Deiner Aufgabe a)
Sei xy>0. Es ist entweder x<0 oder x>0.
Betrachte x>0. Es ist x-1>0.
Mit Axiom 4) folgt aus 0<x-1 und 0<xy, daß 0<x-1(xy)=(x-1x)y = y. Fertig
Wenn x<0, dann ist x-1<0.
Mit Axiom 4) folgt aus x-1<0 und 0<xy, daß x-1(xy) < 0, also y<0. Fertig.

Reicht das für den Anfang?
Gruß
Matroid
Seitenanfangvoriger Beitragnächster BeitragSeitenende Link zu diesem Beitrag

Matroid (Matroid)
Suche alle Beiträge dieser Person in dieser Hauptrubrik
Veröffentlicht am Sonntag, den 26. November, 2000 - 19:19:   Beitrag drucken

Bei 0<xx fehlt noch die Betrachtung des Falles x<0.
Dafür noch eine Hilfsaussage:
Wenn a<0 dann ist 0<(-a)
Beweis: wenn a<0 folgt aus Axiom 3), daß a+(-a)<0+(-a).

Für x<0 kann man also 0<(-x) schreiben.
Dann folgt as 4) wenn 0<(-x) und 0<(-x), dann ist 0<(-x)2.
Nun muß man noch zeigen, daß (-x)2 = x2
Das kann man so sehen, indem man zeigt:
(-a)*b = (-ab) (**)
Beweis: einerseits ist ab + (-ab) = 0
Andererseits ist ab + (-a)b = (a+(-a))b=0b=0
Folglich ist (-ab)=(-a)b
mit a=x und b=-x folgt: (-(x*(-x))) = (-x)*(-x)
und durch nochmalige Anwendung von (**)
(-(-(xx))) = (-x)*(-x)
Das mag's dann gewesen sein.
Seitenanfangvoriger Beitragnächster BeitragSeitenende Link zu diesem Beitrag

Cap23
Suche alle Beiträge dieser Person in dieser Hauptrubrik
Veröffentlicht am Mittwoch, den 29. November, 2000 - 19:37:   Beitrag drucken

Hmm, kommt mir doch alles sehr bekannt vor... =)
Die b) hab ich so gemacht:

I.
4xy <= (x+y)²
4xy <= x² +2xy +y²
0 <= x² -2xy +y²
0 <= (x-y)²
Sei k=x-y dann ist 0 <= k² (s.o.)

II.
(x+y)² <= 2(x²+y²)
x² +2xy +y² <= 2x² + 2y²
0 <= x² -2xy +y² (s.o.)

Hat jemand eine bin. Formel fuer die c) gefunden?
Sowas wie (x+y+z)², was dann irgendwie passt?

Beitrag verfassen
Das Senden ist in diesem Themengebiet nicht unterstützt. Kontaktieren Sie den Diskussions-Moderator für weitere Informationen.

ad

Administration Administration Abmelden Abmelden   Previous Page Previous Page Next Page Next Page